From 52a6ddd9b646bba5f6b169793fd86a2e1af64a1b Mon Sep 17 00:00:00 2001 From: "W. Trevor King" Date: Wed, 1 Jul 2009 08:53:27 -0400 Subject: [PATCH] Added Young and Freedman v12 p21.84 solution --- asymptote/ElectroMag.asy | 20 ++- asymptote/Mechanics.asy | 8 +- .../Young_and_Freedman_12/problem21.84.tex | 122 +++++++++++++++++- 3 files changed, 144 insertions(+), 6 deletions(-) diff --git a/asymptote/ElectroMag.asy b/asymptote/ElectroMag.asy index e66ed09..b8d2775 100644 --- a/asymptote/ElectroMag.asy +++ b/asymptote/ElectroMag.asy @@ -22,10 +22,15 @@ struct Charge { } void draw(picture pic=currentpicture) { - frame picF; + picture picF; + picture picL; + label(picL, L); + pair pLabelSize = 1.2 * (max(picL)-min(picL)); path c = scale(radius)*unitcircle; filldraw(picF, c, fill, outline); - // TODO: label + label(pic = picF, + L = L, + position = (0,-(radius+pLabelSize.y/2))); add(pic, picF, center); } } @@ -44,14 +49,23 @@ Charge nCharge(pair center=(0,0), real q=-1, real radius=2mm, pen outline=curren return c; } +// neutral charge +Charge neutralCharge(pair center=(0,0), real radius=2mm, pen outline=currentpen, Label L="") +{ + Charge c = Charge(center=center, q=0, radius=radius, outline=outline, L=L, fill=grey); + return c; +} + // auto-signed charge Charge aCharge(pair center=(0,0), real q=1, real radius=2mm, pen outline=currentpen, Label L="") { Charge c; if (q > 0) { c = pCharge(center, q, radius, outline, L); - } else { + } else if (q < 0) { c = nCharge(center, q, radius, outline, L); + } else { + c = neutralCharge(center, radius, outline, L); } return c; } diff --git a/asymptote/Mechanics.asy b/asymptote/Mechanics.asy index f3f3e01..046ac5a 100644 --- a/asymptote/Mechanics.asy +++ b/asymptote/Mechanics.asy @@ -199,7 +199,6 @@ struct Angle { } } -// TODO: ihat, ijhat Vector hatVect (string name, pair center=(0,0), real dir=0) { string L = replace("$\mathbf{\hat{X}}$", "X", name); Vector v = Vector(center=center, mag=5mm, dir=dir, L=L, outline=rgb(0,0,0)); @@ -216,6 +215,13 @@ Vector jhat (pair center=(0,0), real dir=90) { return v; } +void draw_ijhat(pair center=(0,0), real idir=0) { + Vector ihat = ihat(center, idir); + Vector jhat = jhat(center, idir+90); + ihat.draw(); + jhat.draw(); +} + // ---------------------- Shapes ------------------------- struct Wire { diff --git a/latex/problems/Young_and_Freedman_12/problem21.84.tex b/latex/problems/Young_and_Freedman_12/problem21.84.tex index 0746d47..b4bef88 100644 --- a/latex/problems/Young_and_Freedman_12/problem21.84.tex +++ b/latex/problems/Young_and_Freedman_12/problem21.84.tex @@ -1,6 +1,6 @@ \begin{problem*}{84} Two tiny balls of mass $m$ carry equal but opposite charges of -magnitude $q$. They are tied to the came ceiling hook by light +magnitude $q$. They are tied to the same ceiling hook by light strings of length $L$. When a horizontal uniform electric field $E$ is turned on, the balls hang with an angle $\theta$ between the strings (Fig.~21.46). Assume that the force one ball exerts on the @@ -8,9 +8,127 @@ other is much smaller than the force exerted by the horizontal electric field. \Part{a} Which ball (the right or the left) is positive, and which is negative? \Part{b} Find the angle $\theta$ between the strings in terms of $E$, $q$, $m$, and $g$. \Part{c} As -the electric field is gradually increased in strength, what does ytour +the electric field is gradually increased in strength, what does your result from \Part{b} give for the largest possible angle $\theta$? \end{problem*} +\begin{nosolution} +\begin{center} +\begin{asy} +import Mechanics; +import ElectroMag; + +real u = 1cm; // Length of 1 m on the page +real L = 2; // Length of cable +real phi = 30; // Half angle between cables +real dy = .6; // Distance below charges to E field vector + +Charge a = neutralCharge(dir(-90+phi)*L*u, L="$q_R$"); +Charge b = neutralCharge(dir(-90-phi)*L*u, L="$q_L$"); +Wire La = Wire((0,0), a.center, L="$L$"); +Wire Lb = Wire(b.center, (0,0), L="$L$"); +Angle theta = Angle(a.center, (0,0), b.center, L="$\theta$"); +Surface s = Surface((a.center.x, 0), (b.center.x, 0)); +Vector E = EField(a.center - (0,dy)*u, mag=(a.center.x - b.center.x), dir=-180, L="$E$"); + +s.draw(); +La.draw(rotateLabel=false); +Lb.draw(rotateLabel=false); +theta.draw(); +E.draw(); +a.draw(); +b.draw(); +\end{asy} +\end{center} +\end{nosolution} + \begin{solution} +\Part{a} +\begin{center} +\begin{asy} +import Mechanics; +import ElectroMag; + +real u = 1cm; // Length of 1 m on the page +real L = 2; // Length of cable +real phi = 30; // Half angle between cables +real dy = .6; // Distance below charges to E field vector + +Charge a = nCharge(dir(-90+phi)*L*u, L="$q_{R-}$"); +Charge b = pCharge(dir(-90-phi)*L*u, L="$q_{L+}$"); +Wire La = Wire((0,0), a.center, L="$L$"); +Wire Lb = Wire(b.center, (0,0), L="$L$"); +Angle theta = Angle(a.center, (0,0), b.center, L="$\theta$"); +Surface s = Surface((a.center.x, 0), (b.center.x, 0)); +Vector E = EField(a.center - (0,dy)*u, mag=(a.center.x - b.center.x), dir=-180, L="$E$"); + +s.draw(); +La.draw(rotateLabel=false); +Lb.draw(rotateLabel=false); +theta.draw(); +E.draw(); +a.draw(); +b.draw(); +\end{asy} + +\begin{asy} +import Mechanics; +import ElectroMag; + +real phi = 30; // Half angle between cables + +Charge a = nCharge(); +Vector Fq = Force(a.center, mag=.4cm, dir=-180, L="$F_q$"); +Vector FT = Force(a.center, mag=1cm, dir=90+phi, L="$F_T$"); +Vector FE = Force(a.center, mag=Sin(phi)*FT.mag+Fq.mag, dir=0, L="$F_E$"); +Vector Fg = Force(a.center, mag=Cos(phi)*FT.mag, dir=-90, L="$F_g$"); +Fq.draw(); +FT.draw(); +FE.draw(); +Fg.draw(); +a.draw(); + +Charge b = pCharge((-2cm, 0)); +Fq.center = FT.center = FE.center = Fg.center = b.center; +Fq.dir += 180; +FT.dir -= 2*phi; +FE.dir += 180; +Fq.draw(); +FT.draw(); +FE.draw(); +Fg.draw(); +b.draw(); + +draw_ijhat((a.center+b.center)/2 - (0,1cm)); +\end{asy} +\end{center} + +Because they have opposite charges, the two balls are attracted to one +another by $F_q$. The combination of gravity $F_g$ and string tension +$F_T$ also works to bring the balls back together. The only remaining +force (which must push the charges apart to balance the previous +attractive forces) is the force from the external electric field +$F_E$. So $F_E$ on the right hand charge $q_R$ must be to the right, +and $F_E$ on the left hand charge must be to the left. Because +$\vect{F}_E=q\vect{E}$, this means that $q_R<0$ and $q_L>0$, so +\ans{the left ball is positive and the right ball is negative}. + +\Part{b} +Balancing forces on the right hand ball (taking care of sign up front) +we have +\begin{align} + \sum F_y &= F_T\cos(\theta/2) - F_g = 0 \\ + F_t &= \frac{F_g}{\cos(\theta/2)} \\ + \sum F_x &= F_E - F_q - F_T \sin(\theta/2) = F_E - F_q - F_g \tan(\theta/2) \approx F_E - F_g \tan(\theta/2) = 0 \\ + F_E &= qE = F_g \tan(\theta/2) = mg \tan(\theta/2) \\ + \theta &= \ans{2 \arctan\p({\frac{qE}{mg}})} +\end{align} +Where we used the fact (given in the problem) that $F_q \ll F_E$ to +ignore $F_q$ in our answer. + +\Part{c} +As $E\rightarrow\infty$, $F_E\rightarrow\infty$, so the balls get +pulled further and further apart until $\ans{\theta=180\dg}$. In +terms of our equation from \Part{b}, $\arctan(+\infty)=90\dg$, so +$\theta=2\cdot90\dg=180\dg$. \end{solution} -- 2.26.2